k is uniformly chosen from the interval (-5,5) . Let p be the probability that the quartic f(x)=kx^4+(k^2+1)x^2+k has 4 distinct real roots such that one of the roots is less than -4 and the other three are greater than -1. Find the value of 1000p.

i think the answer is 6